Формула включения-исключения — различия между версиями

Материал из Викиконспекты
Перейти к: навигация, поиск
(Формула включения-исключения)
(Формула включения-исключения)
Строка 27: Строка 27:
  
  
<center>
+
Тогда исходя из предположения индукции имеем, что
<tex> | B | = \sum \limits_{I \subset \{ 1,2, \ldots ,n-1 \} }  (-1)^{|I|+1}  \Big| \bigcap \limits_{ j \in I } A_j \Big| </tex>
+
<tex> | B | = \sum \limits_{I_{n-1}}  (-1)^{|I_{n-1}|+1}  \Big| \bigcap \limits_{ j \in I_{n-1} } A_j \Big| </tex>
  
  
<tex> | A | = | B | + | A_n | - | B \cap A_n |</tex>
+
Кроме того, так как формула верна для <tex>~l=2</tex> (из базы индукции), то верно равенство <tex> | A | = | B | + | A_n | - | B \cap A_n |</tex>
 
 
  
 +
<center>
 
<tex> \Big| B \bigcap A_n \Big| = \Bigg| \Bigg( \bigcup \limits_{i=1}^{n-1}A_i \Bigg) \bigcap A_n \Bigg|= \Bigg| \bigcup \limits_{i=1}^{n-1} \bigg( A_i \bigcap A_n \bigg) \Bigg| = </tex>
 
<tex> \Big| B \bigcap A_n \Big| = \Bigg| \Bigg( \bigcup \limits_{i=1}^{n-1}A_i \Bigg) \bigcap A_n \Bigg|= \Bigg| \bigcup \limits_{i=1}^{n-1} \bigg( A_i \bigcap A_n \bigg) \Bigg| = </tex>
  

Версия 02:48, 19 октября 2011

Формула включения-исключения

Формула включения-исключения - это комбинаторная формула, которая позволяет определить мощность объединения конечных множеств, если известны их мощности и мощности всех их возможных пересечений.

Случай для двух множеств

Например, в случае двух множеств [math]~A, B[/math] формула включения-исключения имеет вид:

[math] | A \cup B | = | A | + | B | - | A \cap B |[/math]

В сумме [math]~| A | + | B |[/math] элементы пересечения [math]A \cap B[/math] учтены дважды, и чтобы компенсировать это мы вычитаем [math] | A \cap B |[/math] из правой части формулы. Справедливость этого рассуждения видна из диаграммы Эйлера-Венна для двух множеств, приведенной на рисунке справа.

Таким же образом и в случае [math]~n\gt 2[/math] множеств процесс нахождения количества элементов объединения [math]A_1 \cup A_2 \cup \ldots \cup A_n[/math] состоит во включении всего, затем исключении лишнего, затем включении ошибочно исключенного и так далее, то есть в попеременном включении и исключении. Отсюда и происходит название формулы.

Теорема:
Пусть [math] A = \bigcup \limits_{i=1}^{n}A_i [/math] , тогда по формуле включения-исключения:
[math] | A | = \sum \limits_{I_n } (-1)^{k+1} \Big| \bigcap \limits_{ j \in I_n } A_j \Big| [/math]
Причем [math] I_n = (i_1,i_2, \ldots ,i_k) \subset \{ 1,2, \ldots ,n \} [/math], то есть некоторый набор индексов множеств(индексы этих множеств не могут превышать число [math]~n[/math]), пересечение которых мы ищем в текущем слагаемом суммы. За [math] k [/math] принимаем количество таких индексов в текущем [math] I_n [/math], за [math] j [/math] индекс текущего множества (причем [math] j \in I_n [/math]), которое будет входить в пересечение в текущем слагаемом.
Доказательство:
[math]\triangleright[/math]

Будем доказывать теорему, опираясь на метод математической индукции.

Пусть [math]~l[/math] — это количество множеств, мощность пересечения которых мы ищем. Для случая [math]~l=1[/math] и [math]~l=2[/math] теорема, очевидно, верна. Таким образом, [math]~l=2[/math] — база индукции.

Предположим, что для [math]~l=n-1[/math] теорема верна, то есть равенство выполняется. Докажем, что равенство истинно для [math]~l=n[/math]


Пусть [math] A [/math]— пересечение [math]~n[/math] множеств. Тогда очевидно, что [math] A = \bigcup \limits_{i=1}^{n}A_i = \Bigg( {\bigcup \limits_{i=1}^{n-1}A_i} \Bigg) \cup A_n [/math]. Пусть [math] B = \bigcup \limits_{i=1}^{n-1}A_i [/math]


Тогда исходя из предположения индукции имеем, что [math] | B | = \sum \limits_{I_{n-1}} (-1)^{|I_{n-1}|+1} \Big| \bigcap \limits_{ j \in I_{n-1} } A_j \Big| [/math]


Кроме того, так как формула верна для [math]~l=2[/math] (из базы индукции), то верно равенство [math] | A | = | B | + | A_n | - | B \cap A_n |[/math]

[math] \Big| B \bigcap A_n \Big| = \Bigg| \Bigg( \bigcup \limits_{i=1}^{n-1}A_i \Bigg) \bigcap A_n \Bigg|= \Bigg| \bigcup \limits_{i=1}^{n-1} \bigg( A_i \bigcap A_n \bigg) \Bigg| = [/math]


[math] = \sum \limits_{I \subset \{ 1,2, \ldots ,n-1 \} } (-1)^{|I|+1} \bigg| \bigcap \limits_{ j \in I } \Big( A_j \bigcap A_n \Big) \bigg| = \sum \limits_{I \subset \{ 1,2, \ldots ,n-1 \} } (-1)^{|I|+1} \Big| \bigcap \limits_{ j\in I \cup \{ n \} } A_j \Big| [/math]


Таким образом:


[math] | A |=| A_n |+\Bigg( \sum \limits_{I \subset \{ 1,2, \ldots ,n-1 \} } (-1)^{|I|+1} \Big| \bigcap \limits_{ j \in I } A_j \Big| \Bigg) - - \Bigg( \sum \limits_{I \subset \{ 1,2, \ldots ,n-1 \} } (-1)^{|I|+1} \Big| \bigcap \limits_{ j\in I \cup \{ n \} } A_j \Big| \Bigg) = \sum \limits_{I \subset \{ 1,2, \ldots ,n \} } (-1)^{|I|+1} \Big| \bigcap \limits_{ j \in I } A_j \Big| [/math]

[math]\triangleleft[/math]